Docsity
Docsity

Prepare for your exams
Prepare for your exams

Study with the several resources on Docsity


Earn points to download
Earn points to download

Earn points by helping other students or get them with a premium plan


Guidelines and tips
Guidelines and tips

Differential and Integral Calculus - Final Exam Solutions | M 408C, Exams of Mathematics

Material Type: Exam; Class: DIFFEREN AND INTEGRAL CALCULUS; Subject: Mathematics; University: University of Texas - Austin; Term: Spring 2011;

Typology: Exams

2010/2011

Uploaded on 06/28/2011

koofers-user-w3r
koofers-user-w3r 🇺🇸

10 documents

1 / 5

Toggle sidebar

This page cannot be seen from the preview

Don't miss anything!

bg1
Course: Differential and Integral Calculus - M408C Instructor: Gerard Brunick
Final Exam - Solutions
1. (7 pts) Compute d
dx {x2esin(x)}. Please do not simplify your answer.
Solution: 2xesin(x)+x2esin(x)cos(x)
2. (8 pts) Suppose that fis a continuous function and set F(x) = Rx2
0f(y) dy.
Compute F0(x). You will receive more partial credit for an incorrect answer if
you define intermediate functions and clearly indicate the rules that you are using.
Solution: Set G(x) = Rx
0f(y) dy, so G0(x) = f(x), and
F0(x) = d
dxG(x2) = G0(x2)2x= 2xf(x2).
3. (8 pts) Let fbe a differentiable function whose domain is [2,2]. Further suppose
that f(0) = 0 and f0is given by the following graph. Again, you are given the
graph of f0not the graph of f. Please answer the following questions using interval
notation. For example [a, b) {c, d} (e, f]. Don’t worry to much about getting
the numbers exactly right. If you are within 1/4 of the true values, you will receive
full credit.
x
-2 -1 1 2
f0(x)
-1
1
2
3
4
(a) (2 pts) At which of the points in the interval (2,2) is fincreasing?
Solution: (2,3/2).
Page: 1/5
pf3
pf4
pf5

Partial preview of the text

Download Differential and Integral Calculus - Final Exam Solutions | M 408C and more Exams Mathematics in PDF only on Docsity!

Final Exam - Solutions

  1. (7 pts) Compute (^) dxd {x^2 esin(x)}. Please do not simplify your answer. Solution: 2 xesin(x)^ + x^2 esin(x)^ cos(x)
  2. (8 pts) Suppose that f is a continuous function and set F (x) =

∫ (^) x 2 0 f^ (y) dy. Compute F ′(x). You will receive more partial credit for an incorrect answer if you define intermediate functions and clearly indicate the rules that you are using. Solution: Set G(x) =

∫ (^) x 0 f^ (y) dy, so^ G

′(x) = f (x), and

F ′(x) =

d dx

G(x^2 ) = G′(x^2 )2x = 2xf (x^2 ).

  1. (8 pts) Let f be a differentiable function whose domain is [− 2 , 2]. Further suppose that f (0) = 0 and f ′^ is given by the following graph. Again, you are given the graph of f ′^ not the graph of f. Please answer the following questions using interval notation. For example [a, b) ∪ {c, d} ∪ (e, f ]. Don’t worry to much about getting the numbers exactly right. If you are within 1/4 of the true values, you will receive full credit.

x -2 -1 1 2

f ′(x)

(a) (2 pts) At which of the points in the interval (− 2 , 2) is f increasing? Solution: (− 2 , 3 /2).

(b) (2 pts) At which of the points in the interval (− 2 , 2) is f concave up? Solution: The function f is concave up when f ′′^ > 0. These are the points where f ′^ is increasing: (− 3 / 2 , 0). (c) (2 pts) Is f (−1) positive or negative? Solution: It is negative. (d) (2 pts) Does f have a global max? If so, where does it occur? Solution: The function f has global max at x = 3/2 by the first derivative test.

  1. (7 pts) Compute (^) dxd {cos(x) tan-1(x^2 )}. Please do not try to simplify your answer. The function tan-1^ is the inverse of tan not 1/ tan. Solution: − sin(x) tan-1(x^2 ) + 2 x1+^ cos(x 4 x ).
  2. (9 pts) Let f be function with domain [0, 2] and f (x) = x^3 − 6 x^2 + 9x + 1 when 0 ≤ x ≤ 2.

(a) (3 pts) Find the critical points of the function f that lie in the interval (0, 2). Solution: We see that f ′(x) = 3x^2 − 12 x + 9 = 2(x − 1)(x − 3), so the only critical point in the interval is 1. (b) (3 pts) What is the (global) maximum value of f on the interval [0, 2] and where does it occur? Put a circle around the maximum value and put a box around the number in the domain where the maximum occurs. Solution: We just check the critical points and the endpoints: f (0) = 1, f (1) = 5, f (2) = 3. So the maximum value is 5 and it occurs at 1. (c) (3 pts) What is the (global) minimum value of f on the interval [0, 2] and where does it occur? Put a circle around the minimum value and put a box around the number in the domain where the minimum is occurs. Solution: We have already looked at the critical points, so it is clear that the minimum value is 1 and it occurs at 0.

  1. (6 pts) Compute (^) dxd {ln(x^3 )}. Solution: (^) dxd {ln(x^3 )} = 3/x.
  2. (6 pts) Let f be a differentiable and invertible function, set g(x) = f −^1 (x), and
  1. (7 pts) Compute

∫ (^) t

0

x cos(x^2 ) dx. Solution: We make the substitution u = x^2 , so ∫ (^) t

0

x cos(x^2 ) dx =

∫ (^) t 2

0

cos(u) 2

du =

[

sin(u)/ 2

]u=t^2 u=

= sin(t^2 )/ 2.

Set θ = sin−^1 x, so cos θ =

1 − x^2 and tan θ = sin θ/ cos θ = x/

1 − x^2.

  1. (7 pts) Compute

∫ (^) t

0

x sin(x) dx. Solution: We use integration by parts with F (x) = x, f (x) = 1, G(x) = − cos(x) and g(x) = sin(x), so ∫ (^) t

0

x sin(x) dx =

[

−x cos(x)

]x=t x=0 +

∫ (^) t

0

cos(x) dx

=

[

sin(x) − x cos(x)

]x=t x=0 = sin(t)^ −^ t^ cos(t).

  1. (7 pts) Let f (x) = x^3 + 2x^2 and let g(x) = 5 x + 6. Setup the integral that you would use to compute the area of the bounded re- gion that is contained between f and g. You do not need to actually compute this inte- gral, but your final answer should not con- tain “f ” or “g”. It should also not contain any absolute value signs. The following plot is provided for you convenience.

x -4 -3 -2 -1 1 2 3 4

f(x) g(x)

Solution: The area is given by: ∫ (^) − 1

− 3

f (x) − g(x) dx +

− 1

g(x) − f (x) dx

− 3

x^3 + 2x^2 − 5 x − 6 dx +

− 1

−x^3 − 2 x^2 + 5x + 6 dx

  1. (7 pts) Let S be a sphere with radius r. Write down the integral that you would use to compute the vol- ume of the bottom fourth of the sphere. Please do not compute the integral.

Solution: Let h = 0 correspond to the bottom of the sphere, and let f (h) denote the radius of the circular cross section of the sphere which is at the height h. Then f 2 (h) + (r − h)^2 = r^2 , so f 2 (h) = 2h(r − h). This means that the area of the cross section at height h is π 2 hr − h^2 , and the desired integral is:

π

∫ (^) r/ 2

0

2 hr − h^2 dh.

  1. (7 pts) Compute

∫ (^) t

0

3 x^5 sin(x^3 ) dx. You will receive more credit for an incorrect answer if you clearly indicate what you are doing. You do not need to simplify your answer. Solution: We make the substitution u = x^3 followed by integration by parts with with F (u) = u and G(u) = − cos(u) to see that ∫ (^) t

0

x^5 sin(x^3 ) dx =

∫ (^) t 3

0

u sin(u) dx

[

−u cos(u)

]u=t 3 u=

∫ (^) t 3

0

cos(u) du

[

sin(u) − u cos(u)

]u=t^3 u= = sin(t^3 ) − t^3 cos(t^3 ).

  1. (7 pts) Compute

∫ (^) t 0 cos

(^3) (x) dx. You will receive more credit for an incorrect answer if you clearly indicate what you are doing. You do not need to simplify your answer. Solution: Making the change of variable u = sin(x) gives ∫ (^) t

0

cos^3 (x) dx =

∫ (^) t

0

{ 1 − sin^2 (x)} cos(x) dx

∫ (^) sin(t)

sin(0)

1 − u^2 du

[

u − u^3 / 3

]sin(t) sin(0) = sin(t) − sin^3 (t)/ 3.